An electricity and magnetism problem by Spandan Senapati

In an infinite two dimensional square grid of resistors find the equivalent resistance between two nearest diagonal points if each resistance is of π \pi ohms.


The answer is 2.

This section requires Javascript.
You are seeing this because something didn't load right. We suggest you, (a) try refreshing the page, (b) enabling javascript if it is disabled on your browser and, finally, (c) loading the non-javascript version of this page . We're sorry about the hassle.

1 solution

Spandan Senapati
Jun 24, 2018

Assigning points with coordinates (m,n) and the point where current is injected be (0,0) with B being the point (1,1) we wish to find the equivalent resistance between A and B.Assign each of the resistances 1 Ω 1 \Omega so that finally the equivalent resistance can be scaled by a factor of R R .We superpose on this condition another in which current is ejected out from the node B B to get R e q = 2 ( V 1 , 1 V 0 , 0 ) R_{eq}=2(V_{1,1}-V_{0,0}) It follows from Kirchoff's current law that 4 V m , n = V m , n + 1 + V m , n 1 + V m + 1 , n + V m 1 , n 4V_{m,n}=V_{m,n+1}+V_{m,n-1}+V_{m+1,n}+V{m-1,n} Assuming a solution V m , n = A ( α , β ) ϕ m ψ n V_{m,n}=A(\alpha ,\beta )\phi ^m\psi^n we have 4 ( ϕ + 1 ϕ + ψ + 1 ψ ) = 0 4-(\phi +\frac {1}{\phi }+\psi +\frac {1}{\psi })=0 letting ϕ = e i α , ψ = e i β \phi =e^{i\alpha },\psi =e^{i\beta } it becomes 4 ( e i α + e i α + e i β + e i β ) = 0 2 cos α cos β = 0 4-(e^{i\alpha }+e^{-i\alpha }+e^{i\beta }+e^{-i\beta })=0\implies 2-\cos \alpha -\cos \beta =0 hence V m , n = A ( α , β ) e i ( α m + β n ) V_{m,n}=A(\alpha ,\beta )e^{i(\alpha m+\beta n)} also it follows from physical symmetry that V m , n = V m , n = V m , n = V m , n = A ( α , β ) e i ( m α + n β ) V_{m,n}=V{m,-n}=V{-m,-n}=V_{-m,n}=A(\alpha ,\beta )e^{i(|m|\alpha +|n|\beta )} hence the general solution is given by the superposition over a period ( π , π ) (-\pi,\pi ) , V m , n = π π A ( α , β ) e i ( m α + n β ) V_{m,n}=\int _{-\pi }^{\pi }A(\alpha ,\beta )e^{i(|m|\alpha +|n|\beta )} further notice that I m , 0 = 0 I_{m,0}=0 so 4 V m , 0 V m , 1 V m , 1 V m + 1 , 0 V m 1 , 0 = 0 π π 2 i A ( α , β ) sin β e i m α d α = 0 4V_{m,0}-V_{m,1}-V{m,-1}-V_{m+1,0}-V{m-1,0}=0\implies -\int _{-\pi }^{\pi }2iA(\alpha ,\beta )\sin \beta e^{im\alpha }d\alpha =0 Recalling the Fourier series expansion of a function f ( t ) = n = c n e i n ω t , c n = 1 T T / 2 T / 2 f ( t ) e i n ω t d t f(t)=\sum _{n=-\infty }^{\infty }c_{n}e^{in\omega t },c_{n}=\frac {1}{T}\int _{-T/2}^{T/2}f(t)e^{-in\omega t}dt for real values functions since the i sin i\sin terms sum to zero.Hence it follows that \sum _I_{m,0}e^{im\alpha }=I_{0,0}=-1 where we assume 1 A 1 A current is injected into the node at A A hence A ( α , β ) = 1 4 π i sin β A(\alpha ,\beta )=\frac {1}{4\pi i\sin \beta } ,this gives us the function V m , n V{m,n} the equivalent resistance between the nodal points 0 , 0 0,0 and m , n m,n , m , n > 0 m,n>0 is hence R m , m = 1 2 π i π π e i ( m α + n β ) 1 sin β d α R_{m,m}=\frac {1}{2\pi i}\int _{-\pi }^{\pi }\frac {e^{i(m\alpha +n\beta )}-1}{\sin \beta }d\alpha split π π = π 0 + 0 π \int _{-\pi }^{\pi }=\int _{-\pi }^0+\int _{0}^{\pi } and use De-Moivre's formula with change of variable in π 0 \int _{-\pi }^{0} integral to get R m , n = 1 π i 0 π 1 cos m α e i n β sin β d α R_{m,n}=\frac {1}{\pi i}\int _{0}^{\pi }\frac {1-\cos m\alpha e^{in\beta }}{\sin \beta }d\alpha it follows by integration that R 1 , 1 = 2 R π R_{1,1}=\frac {2R}{\pi } and also yields the general relation R m , m = 2 R π k = 1 n 1 2 k 1 R_{m,m}=\frac {2R}{\pi }\sum _{k=1}^{n}\frac {1}{2k-1}

The question should be a decimal OK type question. :D. Theen none will search for integers

Md Zuhair - 2 years, 11 months ago

What the hell!

Swapnil Das - 2 years, 7 months ago

0 pending reports

×

Problem Loading...

Note Loading...

Set Loading...